Уравнения метрического поля для действия Жордана-Бранса-Дикке

Учитывая действие Джордана-Бранса-Дикке:

С "=" д 4 Икс г ( ф р + ю ф ( ф ) 2 + л м ( ψ ) ) .

Я пытался получить уравнения метрического поля, варьируя метрику, и получил следующее:

1 2 г мю ν р + р мю ν + ю ф 2 [ 1 2 г мю ν ( ф ) 2 + мю ф ν ф ] 1 2 ф г мю ν л м ( ψ ) "=" 0

я менял условия г , р мю ν , г мю ν и мю ф ν ф г мю ν . Если нас интересуют только уравнения метрического поля, то так ли это? Если бы мне нужны были уравнения для гравитационного поля, нам пришлось бы варьировать метрику и поле. ф верно?

РЕДАКТИРОВАТЬ: по второму правилу Лейбница я считал:

α α ( г мю ν ф дельта г мю ν ) "=" г мю ν α α ( ф ) дельта г мю ν г мю ν α ( ф ) α ( дельта г мю ν ) г мю ν α ( ф ) α ( дельта г мю ν ) г мю ν ф α α ( дельта г мю ν )

Я вытащил метрику, поэтому мне не нужно иметь дело с 6 терминами. Те, которые нам нужны, являются только первым и вторым в правой части этого уравнения.

Что случилось с термином /phi R??
Я разделил все уравнение на ф и это будет первый термин, который появится
Это неправильно. Приходится интегрировать по частям.
Вы уже применили первую ковариантную производную. Вы должны сделать это еще раз с другой ковариантной производной.
Что ты имеешь в виду? Я уже применил оба. Что я путаю где?
Вы применили их оба одновременно.
Я обратился α первым г мю ν ф дельта г мю ν а потом α на 2 члена, которые мы получаем из первой ковариантной производной, поскольку один из них является ковариантной производной метрического тензора, что дает мне 4 члена.
Проверьте мой ответ здесь, может помочь: physics.stackexchange.com/questions/128501/…

Ответы (1)

The дельта ( ф р ) срок будет:

дельта ( ф р ) "=" дельта ( ф г мю ν р мю ν ) "=" ф дельта г мю ν р мю ν + ф дельта р мю ν г мю ν

Термин: ф дельта г мю ν р мю ν готово, здесь вариация обратного метрического тензора уже является множителем. Теперь второй термин:

ф дельта р мю ν г мю ν "=" ф ( г мю ν мю ν ) дельта г мю ν

где я использовал личность Палатини. Теперь у нас есть, например, для термина коробки:

ф г мю ν дельта г мю ν "=" ф г мю ν α α дельта г мю ν "=" α ( ф г мю ν α дельта г мю ν ) α ф г мю ν α дельта г мю ν

Первый член представляет собой полную производную. Мы будем игнорировать его как граничный член. Теперь снова воспользуемся правилом Лейбница:

α ф г мю ν α дельта г мю ν "=" α α ( г мю ν ф дельта г мю ν ) + г мю ν дельта г мю ν α α ( ф )

где я использовал метрическую совместимость. Итак, у нас есть:

ф г мю ν дельта г мю ν "=" г мю ν дельта г мю ν α α ( ф ) "=" г мю ν дельта г мю ν ф
Ту же процедуру нужно проделать для двух ковариантных производных. Остальные термины кажутся правильными.

Проблема здесь в том, что скаляр Риччи связан с ф . Когда я впервые столкнулся с такими условиями связи, у меня была та же проблема. В контексте общей теории относительности действие равно:

С "=" д 4 Икс г р .

Вариация приводит к термину г мю ν дельта р мю ν . Мы можем показать, что этот член является полным производным членом, и сократить его. В контексте Бранса Дикке (или других геометрических модификаций гравитации Эйнштейна, ф ( р ) например, Хорндески или поля материи, не минимально связанные с гравитацией) этот термин больше не является полной дивергенцией. Здесь этот термин: ф дельта р мю ν г мю ν . ф усложняет ситуацию, мы не можем теперь отбросить этот термин как таковой, это не тотальный производный термин. Таким образом, мы следуем процедуре, которую я описал выше.

Что касается второй части вопроса, да, вы должны варьироваться и в отношении ф . Здесь ф это не поле материи, это геометрическая величина.

Спасибо! Во всех вариантах действия, которые я сделал до сих пор, R не было связано с каким-либо другим полем, поэтому я просто делаю это быстро, заявляя о тождестве Палатини и теореме Гаусса и говорю, что оно равно нулю.
Я понимаю вашу ситуацию на 100%. Был там, сделал то же самое.
Только одно, разве нет г мю ν отсутствует в 4-м уравнении, которое вы написали на 1-м члене RHS?
Конечно!! Спасибо, что указали на это!
Извините, не могли бы вы уточнить, как вы выполнили второе правило Лейбница? Я получаю 4 термина, когда распределяю две ковариантные производные для этого термина: α α ( г мю ν ф дельта г мю ν ) Учитывая совместимость метрик, я взял метрику напрямую и получил остальные 4 термина, 2 из них компенсируют друг друга?
α действует на вариацию. Я передвинул эту производную перед всеми членами и вычел г мю ν дельта г мю ν ф (правило Лейбница).
Чтобы продолжить, вам нужно убрать полный производный член в первом правиле Лейбница.
Да, я сделал это. Для вашего первого правила Лейбница я рассматривал термин α ( ф г мю ν α ( дельта г мю ν ) ) и распространил α на трех членах, и один из них исчез из-за метрической совместимости, и перестановка этого уравнения привела меня к вашему первому правилу Лейбница. Я проигнорировал термин, который имеет полную производную. Я попробовал ту же процедуру, используя α α ( г мю ν ф дельта г мю ν ) но есть дополнительные термины
Как?? Пожалуйста, напишите эти условия.
я напишу их
Может ли кто-нибудь явно сделать изменение в отношении ф ?
Почему бы вам не задать вопрос по теме, чтобы кто-то мог вам помочь?